LSAT and Law School Admissions Forum

Get expert LSAT preparation and law school admissions advice from PowerScore Test Preparation.

 Administrator
PowerScore Staff
  • PowerScore Staff
  • Posts: 8916
  • Joined: Feb 02, 2011
|
#23088
Complete Question Explanation

Method of Reasoning. The correct answer choice is (A)

The argument is that mainstream economic theory is wrong when it asserts that manufacturers simply respond to the needs and desires of consumers, because most major manufacturers manipulate and create consumer demands.

The stimulus also contains the accusation that mainstream economic theorists must have questionable motives, since they watch television. The stimulus asserted that anyone who watches television knows that manufacturers manipulate and create demand, so the implication is that economic theorists mislead people and have questionable motives.

Answer choice (A): This is the correct answer choice. Whether you believed the main conclusion was that theorists are deliberately misleading people, or simply that mainstream theory is wrong, the claim that manufacturers manipulate and create demands is an initial premise that supports the conclusion.

Answer choice (B) The claim was a premise, not the conclusion, and this choice is wrong. Furthermore, claims about fact are generally supportive, not conclusions.

Answer choice (C) The claim is offered as evidence for the argument's position, so cannot be the position argued against.

Answer choice (D) The claim is evidence for the conclusion, not a potential objection against it.

Answer choice (E) "Supplementary" implies that the information would be extra, possibly unnecessary. However, the claim is central to the argument, so is neither "supplementary" nor "background."
 ellenb
  • Posts: 260
  • Joined: Oct 22, 2012
|
#14294
Dear Powerscore,

For this question is the conclusion implied? I know that the conclusion is that "the economic theory is wrong" however, it is not explicitly stated anywhere in the stimulus.

Thanks in advance!

Ellen
 Nikki Siclunov
PowerScore Staff
  • PowerScore Staff
  • Posts: 1362
  • Joined: Aug 02, 2011
|
#14301
Hi Ellen,

Sorry for the delay in responding to your question.

I'm afraid you mischaracterized the conclusion of the argument. The main point is not that the mainstream theory is wrong. This is implied, of course, by the observation that manufacturers manipulate and create consumer demand, suggesting that they don't simply respond to the needs of consumers. The author, however, has a bigger fish to fry: s/he is more concerned with the motives behind advancing such a flawed theory, not merely with the fact that the theory is wrong. Since the economists advancing it themselves watch television, the author suspects that their motive is something other than concern for scientific truth (i.e. some other, questionable motive). We don't know what that motive is, and we don't really care to find out.

The conclusion can be found in the second clause of the last sentence in the stimulus ("...their motive in advancing...").

The correct answer choice is (A), because the statement in question is a premise for the conclusion that economists have ulterior motives for advancing a flawed economic theory.

Let me know if this helps!

Thanks.
 bk1111
  • Posts: 103
  • Joined: Apr 22, 2017
|
#46331
Is “background information” not considered a premise in a way? I was debating between A and E because of this reason. Going forward, should I assume background information does not support the main conclusion?
 Adam Tyson
PowerScore Staff
  • PowerScore Staff
  • Posts: 5153
  • Joined: Apr 14, 2011
|
#47158
Background information might shed some light on another claim, bk1111, perhaps by framing the scope of the argument or by showing why the issue at hand is being discussed in the first place, but it is not a premise, and is typically not essential to the argument. That answer is only going to be good if the claim in question is neither a premise nor a conclusion of the argument, but something else.
 Jon Denning
PowerScore Staff
  • PowerScore Staff
  • Posts: 904
  • Joined: Apr 11, 2011
|
#48121
Post from another user:

Method of Reasoning (Argument Part)

Is the subsidiary conclusion 'even mainstream economic theorists watch television'?
So the argument flows with introductory information, then the premise that most major manufacturers manipulate, which leads to the subsidiary conclusion that even mainstream economic theorists watch television.
 Jon Denning
PowerScore Staff
  • PowerScore Staff
  • Posts: 904
  • Joined: Apr 11, 2011
|
#48239
I wouldn't describe the clause "even mainstream economic theorists watch television" as an intermediate or subsidiary conclusion, as there's no evidence given in an attempt to prove that statement (the requirement of any conclusion, intermediate or main). Instead that phrase is simply used to apply the preceding information about "as anyone who watches television knows" to mainstream economic theorists, and thereby demonstrate that their motive for the claim in the first sentence isn't purely disinterested concern for scientific truth.

In other words, the economic theorists should know that most major manufacturers manipulate and even create consumer demand, since all tv viewers know this and econ theorists are undoubtedly tv viewers. That phrase simply lumps econ theorists into the tv viewer category, about which we've been told something. And then once we know that the theorists are aware of manufacturers' behavior we're given the conclusion: theorists' motives for their theory are more than just indifferent concern for truth.

I hope that helps!

Also, there's a more comprehensive breakdown of this entire question here (if interested): lsat/viewtopic.php?f=675&t=9045&p=14301
 menkenj
  • Posts: 116
  • Joined: Dec 02, 2020
|
#84069
I had a really difficult time with identifying the conclusion. I was thinking that the final sentence was the main conclusions and that the second sentence was support. When I apply the conclusion test, it is still confusing.

"Economic theorists motives must be something other than disinterested concern for scientific truth, therefore most major manufacturers manipulate and create consumer demand." This just doesn't make sense.

The alternative does make sense to me. "Most major manufacturers manipulate and create consumer demand, therefore economic theorists motives must be something other than disinterested concern for scientific truth."

Why is it that the conclusion test failed me here? What am I doing wrong?
 menkenj
  • Posts: 116
  • Joined: Dec 02, 2020
|
#84070
Nevermind. I thought at first Jon was saying that the second sentence was the conclusion. Phew. I'm on track.

If the question stem has referenced a premise that stated something like, "tv advertisements account for 80% of most people's advertising exposure," would this count as supplementary background info and make answer choice E correct?

Thanks!
 Adam Tyson
PowerScore Staff
  • PowerScore Staff
  • Posts: 5153
  • Joined: Apr 14, 2011
|
#84139
Possibly, menkenj, although if it is a premise then it cannot just be background info. A premise is a central part of an argument, while supplemental background info would be only material that "sets the stage" for the argument while contributing nothing to it. The first sentence of this stimulus fits that bill, actually.

Get the most out of your LSAT Prep Plus subscription.

Analyze and track your performance with our Testing and Analytics Package.